You are on page 1of 15

9 Weyls Theorem

We will give some preliminary material before introducing Weyls Theorem and its proof. In
particular, the following result is fairly straightforward, but nevertheless it leads to some nice
examples.
Theorem 23. Let k be an integer. Then
1
2

(
0
eikx dx =
1

if k =
6 0
if k = 0.

Equivalently,
i2kx

e
0

(
0
dx =
1

if k =
6 0
if k = 0.

(
0
=
n

if k =
6 0
if k = 0.

Also, if n > |k|, then


n1
X

ei2kj/n

j=0

We give a few examples of the usefulness of Theorem 23. We do this by posing problems and
demonstrating a solution to each based on the above theorem. It should be kept in mind that we do
not mean to imply the the following solutions are the most elegant.
Example 1 (Putnam A-5, 1985): Let
Z 2
Im =
cos(x) cos(2x) cos(3x) cos(mx) dx.
0

For which integers m, 1 m 10, is Im 6= 0?


Solution: Recall that
cos x =

eix + eix
.
2

Hence,
Z
Im =
0

m  ikx
2 Y

k=1

+ eikx
2

1
dx = m
2

X
1 ,...,m {1,1}

ei(1 +22 ++mm )x dx.

From Theorem 23, we deduce that each of the integrals in the sum is 0 unless 1 +22 + +mm =
0. Also, if an integral in the sum is not zero, then it is positive. Therefore, Im 6= 0 if and only if there
exist 1 , . . . , m {1, 1} such that 1 +22 + +mm = 0. Note that if 1 +22 + +mm = 0,
then
m(m + 1)
0 1 + 22 + + mm 1 + 2 + + m
(mod 2)
2
so that
m 0 or 3 (mod 4).
34

Thus, Im = 0 for m {1, 2, 5, 6, 9, 10}. To see that the answer is the remaining m, that is
m {3, 4, 7, 8}, observe that
1+23=0

and 1 2 3 + 4 = 0

and if
1 + 22 + + mm = 0,
then
1 + 22 + + mm (m + 1) + (m + 2) + (m + 3) (m + 4) = 0.
Note that in general Im 6= 0 if and only if m 0 or 3 (mod 4).
Example 2 (Putnam A-6, 1985): If p(x) = a0 + a1 x + + am xm is a polynomial with real
coefficients ai , then set
(p(x)) = a20 + + a2m .
Let f (x) = 3x2 + 7x + 2. Find, with proof, a polynomial g(x) with real coefficients such that
(i) g(0) = 1, and
(ii) (f (x)n ) = (g(x)n ) for all positive integers n.
Solution: Observe that Theorem 23 implies
(p(x)) = a20 + + a2m
Z 1


=
a0 + a1 ei2x + + am ei2(mx) a0 + a1 ei2x + + am ei2(mx) dx
0
1

p(ei2x )p(ei2x ) dx.

=
0

Thus,
(f (x)n ) = ((3x + 1)n (x + 2)n )
Z 1
=
(3ei2x + 1)n (ei2x + 2)n (3ei2x + 1)n (ei2x + 2)n dx
0

(3ei2x + 1)n (ei2x + 2)n (ei2x )n (3ei2x + 1)n (ei2x + 2)n (ei2x )n dx

=
0

Z
=

g(ei2x )n g(ei2x )n dx = (g(x)n ),

where g(x) = (3x + 1)(2x + 1) = 6x2 + 5x + 1. Since g(0) = 1, this completes the solution.
Example 3: Let R be a rectangle which is partitioned as a disjoint union (excluding common
edges) of rectangles R1 , . . . , Rn each having sides parallel to the sides of R. Prove that if each Rj
has at least one side of integer length for j = 1, 2, . . . , n, then so does R.
35

Solution: We do not use Theorem 23 directly here but rather the following which is of a similar
flavor:
Suppose and u are real numbers. Then is an integer if and only if
Z u+



i2x

= 0.
e
dx


u

Position the rectangles so that their sides are parallel to the x and y-axes and so that the lower
left corner of R is (0, 0). Suppose j and j are the horizontal and vertical dimensions of Rj ,
respectively, for j = 1, 2, . . . , n. Let (uj , vj ) be the lower left corner of Rj . Then since either j is
an integer or j is an integer, we get from the above that
Z
Z
Z

vj +j Z uj +j
uj +j
vj +j





i2(x+y)
i2x
i2y
e
dx
dy
=
e
dx
e
dy



= 0.
vj





uj
uj
vj
On the other hand, if and are the horizontal and vertical dimensions of R, respectively, then

Z
Z
Z Z




i2(x+y)
i2x
i2y



=
e
dx
e
dx
e
dy




0

n ZZ

n Z
X

X vj +j Z uj +j




i2(x+y)
i2(x+y)
=
e
dx =
e
dx dy = 0.



Rj
vj
uj
j=1

j=1

R

R



Hence, either 0 ei2x dx = 0 or 0 ei2y dy = 0, which implies that either or is an integer,
completing the proof.
We turn now to the main topic of this section. For real, let {} denote the fractional part
of . Note that {} [0, 1) for all . Thus, for example, {2.341} = 0.341, {22/7} = 1/7, and
{22/7} = 6/7. Let {1 , 2 , . . . } be a sequence of real numbers. We say that the sequence is
uniformly distributed modulo one if for every a and b with 0 a b 1,
|{r n : {r } [a, b]}|
= b a.
n
n
lim

The following result is due to Weyl.


Theorem 24. If is a real irrational number, then the sequence {r}
r=1 is uniformly distributed
modulo one.
Before proving Weyls Theorem, we discuss some preliminaries. First, we observe that determining whether a sequence is uniformly distributed may not even be intuitively clear. For

example, suppose R is irrational. Does it follow that {r }


r=1 is uniformly distributed modulo
one? One can see that this is not the case by considering = 2 since
r
n1
|{r n : { 2 } [0, 1/3]}|
.
2
36


Is this an unfair example since = 2 has the property thatsome power of is an integer? To
partially answer this question, we show that for = (1 + 5)/2, the sequence {r }
r=1 is not
uniformly distributed modulo one. Define
!r
!r
1 5
1+ 5
+
for r Z+ {0}.
ur =
2
2
Note that u0 = 2, u1 = 1, and for r 2,
!r
!r
1+ 5
1 5
ur =
+
2
2
=

!r2
1+ 5
2

!2
1+ 5
+
2

!r2
1 5
2

!r2
1+ 5
2

!
3+ 5
+
2

!r2
1+ 5
2

!
1+ 5
1+
+
2

!r2
1 5
2

!2
1 5
2
!
3 5
2

!r2
1 5
2

!
1 5
1+
2

= ur2 + ur1 .

r
Therefore, ur Z+ for all r 0. Note that (1 5)/2 (1, 0) so that (1 5)/2 tends to
zero as r tends to infinity. Therefore,

!
1 + 5 2r+1
lim
=0
r

2
and

!
1 + 5 2r
lim
= 1.
r

This easily implies that


|{r n : {r } [1/4, 3/4]}|
=0
n
n
so that {r }
r=1 is not uniformly distributed modulo one.
We note that it can be shown that, for almost all real numbers , the sequence {r }
r=1 is
uniformly distributed modulo 1. On the other hand, no explicit example of such an is known.
POur proof of Weyls Theorem will depend on the use of Ces`aro sumability. We say that a series
aro sum S if
j=0 aj has Ces`
Pn Pr
r=0
j=0 aj
lim
= S.
n
n+1
We first make an important connection between Ces`aro sums and our usual notion of sums. The
next result implies that if a series converges to S, then its Ces`aro sum is also S; on the other hand,
its Ces`aro sum may exist even if the series diverges.
lim

37

Pr
Theorem 25. Let {aj }
j=0 be a sequence of complex numbers, and set sr =
j=0 aj for r
+
Z {0}. Then
Pn
sr
(i) if limr sr = S, then limr r=0 = S, and
n+1
Pn
sr
j(j+1)/2
(ii) if aj = (1)
, then limr sr does not exist and limr r=0 = 0.
n+1
Proof. (i) Suppose limr
P sr = S. Let  > 0. Then there is an R such that if r R, then
|sr S| < /2. Let A = R
r=0 |sr S|. Let N max{R, 2A/}. Then for n N ,


Pn

n
X

r=0 sr

1




n + 1 S = n + 1 (sr S)
r=0
R

n


X

1
1 X



(sr S)
=
(sr S) +

n+1

n+1
r=0

r=R+1

1
1  

A + (n + 1)

n + (n + 1)
n+1
2
n+1
2
2


< + = .
2 2


Thus,

Pn

sr
= S.
r n + 1
(ii) Here, {aj }
j=0 = {1, 1, 1, 1, 1, 1, 1, 1, . . . } so that s0 = 1, s1 = 0, s2 = 1, s3 = 0,
s4 = 1, . . . . Hence, for every r 0,
n
X
|
sr | 1.
r=0

lim

r=0

Thus,

Pn
lim

sr
= 0.
n+1
r=0

Clearly, limr sr does not exist.


1
3

1
Before continuing, we consider
Pr some examples. First, we show that the Ces`aro sum of 1 + 2 +
+ is infinite. Let sr = j=0 (1/(j + 1)). Note that
Z r+2
1
sr
dt = log(r + 2).
t
1

Thus,
n X
r
X

aj =

r=0 j=0

n
X

sr

r=0

n+2

n
X

log(r + 2)

r=0

n+2
log t dt = t log t 1 = (n + 2) log(n + 2).

38

Hence,

Pn

Pr

aj

(n + 2) log(n + 2)
.
n+1
n+1
Since this last expression tends to infinity with n, we deduce that the Ces`aro sum of 1 + 12 + 13 +
is infinite.
P
As another
example,
we
show
that
it
is
possible
to
have
a
series
j=0 aj with all of its partial
Pr
sums sr = j=0 aj bounded and with no Ces`aro sum. We take

if j = 22k where k Z+ {0}


1
aj = 1 if j = 22k1 where k Z+
.

0
otherwise
r=0

j=0

Then we get that a0 , a1 , a2 , . . . is 0, 1, 1, 0, 1, 0, 0, 0, 1, . . . , and s0 , s1 , s2 , . . . is 0, 1, 0, 0, 1,


1, 1, 1, 0, . . . . In general,
(
1 if j [22k , 22k+1 ) Z where k Z+ {0}
sj =
.
0 otherwise
Note that for any positive integer m, 1 + 2 + 22 + + 2m < 2m+1 so that
!
2k 1
2X
k1
2k4
X
X
sj =
22j
2j + 22k2
j=0

j=0

j=0

3
22k3 + 22k2 = (22k ).
8
Also,
22k+1
X1
j=0

sj =

k
X

1
22j 22k = (22k+1 ).
2
j=0

P


P

n
n
Thus, infinitely often
a
/(n
+
1)

3/8
and
infinitely
often
a
/(n + 1) 1/2.
j
j
j=0
j=0
This easily implies that the Ces`aro sum cannot exist.
To prove Weyls Theorem (Theorem 24), we will use a little material from Fourier Analysis
which we introduce here. The basic idea is to write a function f (x), which maps the real numbers
to the complex numbers, in the form
f (x) =

f(r)eirx .

(11)

r=

Here, we wish to find numbers f(r) for which the above holds. Since eirx = cos(rx) + i sin(rx),
the series on the right in (11) is often referred to as a trignometric series. It is not always possible
to obtain a trignometric series representation for f (x), but lets suppose for the moment that f (x)
can be expressed in the form (11). We temporarily ignore rigor. Using Theorem 23, observe that
for n |r|,
!
Z 2 X
n Z 2
n
X
1
1
f(r) =
f(j)ei(jr)x dx =
f(j)eijx eirx dx.
2 j=n 0
2 0
j=n
39

Letting n tend to infinity, we deduce that


1
f(r) =
2

f (x)eirx dx.

(12)

Thus, using (12), it is seemingly easy to write a function in the form (11). However, we still need
to discuss when the series in (11) converges. We will require certian conditions on f (x). They are:
(i) f (x) is continuous over the reals.
(ii) f (x + 2) = f (x) for all real x.
The latter condition shouldnt be surprising since the values of f(r) determine f (x) by (11) and
they only depend on the values of f (x) for x [0, 2] by (12). Before continuing, it is worth
noting that (i) and (ii) are not sufficient to imply the convergence of the series in (11). On the other
hand, we will want to consider f (x) in this generality. To deal with this difficulty, we prove a
result of Fejer that with f (x) satisfying (i) and (ii), the series in (11) has Ces`aro sum f (x) (so that
convergence of the series will not be necessary).
Define f(r) by (12), and set
!
j
n
1 X X
n (f, x) =
f (r)eirx
n + 1 j=0 r=j
=


1 
(n + 1)f(0) + nf(1)eix + nf(1)eix +
n+1

n
X
n + 1 |r|
=
f (r)eirx .
n+1
r=n

We will now prove


Theorem 26. If f (x) satisfies (i) and (ii), then n (f, x) converges to f uniformly on R.
We begin with a relationship between n (f, x) and
n
X
n + 1 |r| irx
Kn (x) =
e .
n
+
1
r=n

Observe that
n
X
n + 1 |r|
n (f, x) =
f (r)eirx
n
+
1
r=n

Z 2

n
X
n + 1 |r| 1
irt
=
f (t)e
dt eirx
n
+
1
2
0
r=n
1
=
2

1
=
2

n
X
n + 1 |r| ir(xt)
f (t)
e
dt
n+1
r=n

f (t)Kn (x t) dt.
0

40

Note that in addition to (ii), we have that Kn (x + 2) = Kn (x) for all real x. Letting y = x t
gives
Z
1 x2
n (f, x) =
f (x y)Kn (y) dy
2 x
Z x
Z 2
1
1
f (x y)Kn (y) dy =
f (x y)Kn (y) dy.
=
2 x2
2 0
We now proceed with two lemmas.
Lemma 1. Let x [0, 2). Then Kn (0) = n + 1, and if x 6= 0, then

2
1
sin((n + 1)x/2)
Kn (x) =
.
n+1
sin(x/2)
Proof. Clearly,
n
X
1
n + 1 |r|
Kn (0) =
=
n+1
n+1
r=n

n+1+2

n
X

!
r

= n + 1.

r=1

Now, suppose x (0, 2). Then


n
X
n + 1 |r| irx
Kn (x) =
e
n
+
1
r=n

1
=
n+1
Note that

n
X

inx

(n + 1)e

i(r+n)x

r=n

n
X

i(r+n)x

r=n

Also,

2n
X

eirx =

r=0
n
X
r=1

eirx =

n
X

!
re

irx

+ re

irx

r=1

ei(2n+1)x 1
.
eix 1

ei(n+1)x 1
1
eix 1

so that by taking derivatives, we obtain


i

n
X

reirx =

r=1

Thus,

n
X

(eix 1)i(n + 1)ei(n+1)x (ei(n+1)x 1)ieix


.
(eix 1)2

reirx =

r=1

and

n
X
r=1

reirx =

nei(n+2)x (n + 1)ei(n+1)x + eix


(eix 1)2

nei(n+2)x (n + 1)ei(n+1)x + eix


.
(eix 1)2
41

Hence,
Kn (x) =

1
(n + 1)einx (eix 1)(ei(2n+1)x 1)
(n + 1)(eix 1)2


nei(n+2)x (n + 1)ei(n+1)x + eix neinx (n + 1)ei(n1)x + eix


1
i(n+2)x
inx
ix
e
+
e

2e
(n + 1)(eix 1)2


eix
i(n+1)x
i(n+1)x
e

2
+
e
(n + 1)(eix 1)2


eix
i(n+1)x/2
i(n+1)x/2 2
e

e
(n + 1)(eix 1)2

2
1
(eix/2 eix/2 )2 ei(n+1)x/2 ei(n+1)x/2
n+1
 ix/2
2  i(n+1)x/2
2
1
e
eix/2
e
ei(n+1)x/2
=
n+1
2i
2i

2
sin((n + 1)x/2)
1
=
.
n+1
sin(x/2)
=

This completes the proof.


Lemma 2. Kn (x) has the following properties:
(a) Kn (x) 0 for all real numbers x.
(b) For every > 0 and  > 0, there is an N = N (, ) such that if n N and x (, 2 ),
then |Kn (x)| < .
(c)

1 R 2
Kn (x) dx = 1.
2 0

Comment: (b) is simply asserting that for all > 0, Kn (x) approaches 0 uniformly on (, 2 ).
Proof. Since Kn (x+2) = Kn (x), (a) follows from Lemma 1. For (b), note that for x (, 2)
and for n sufficiently large (independent of x), Lemma 1 implies
Kn (x)

1
1

< .
2
(n + 1) sin (x/2)
(n + 1) sin2 (/2)

Finally, we deduce (c) from


Z 2
Z 2 X
n
1
1
n + 1 |r| irx
Kn (x) dx =
e dx
2 0
2 0 r=n n + 1
Z 2
Z 2
n
X
n + 1 |r| 1
1
irx
=
e dx =
dx = 1,
n + 1 2 0
2 0
r=n
completing the proof.
42

We now give the basic idea behind the proof of Theorem 26. Note that Lemma 2 implies that
for > 0 fixed and small and for n large,
Z 2
Z 2
1
1
Kn (y) dy 0 and
Kn (y) dy = 1
2
2 0
so that
1
2

Kn (y) dy 1.

Also,
1
2
so that
1
n (f, x) =
2

Z
0

f (x y)Kn (y) dy 0

1
f (x y)Kn (y) dy
2

f (x y)Kn (y) dy.

Since f (x) is continuous, f (x y) f (x) for y [, ]. Thus,


 Z

Z
1
1
f (x)Kn (y) dy = f (x)
Kn (y) dy f (x).
n (f, x)
2
2
We now make the above ideas rigorous.
Proof of Theorem 26. We use that f (x) is bounded and uniformly continuous on the compact interval [0, 4]. Thus, there is an M such that |f (x)| M for all x [0, 4]; and for all  > 0, there
is a > 0 such that if x and y are in [0, 4] with |x y| < , then |f (x) f (y)| < . By property
(ii), |f (x)| M for all x R and for all  > 0, there is a > 0 such that |f (x) f (y)| <  for all
x and y in R with |x y| < .
Let  > 0. Let (0, ) such that |f (x) f (y)| < /2 for all x and y in R with |x y| < .
By Lemma 2 (b), there is an N = N (, ) such that if n N and x (, 2 ), then |Kn (x)|
/(4M ). By Lemma 2 (a),
Z

|Kn (y)| dy =

Z
Kn (y) dy

|Kn (y)| dy.


0

43

Thus, we get from Lemma 2 (c) that for all n N ,


Z 2

 Z 2

1

1

|n (f , x) f (x)| =
f (x y)Kn (y) dy
Kn (y) dy f (x)
2 0
2 0
Z 2

1

=
(f (x y) f (x)) Kn (y) dy
2 0
Z

Z 2




1
1

(f (x y) f (x)) Kn (y) dy +
(f (x y) f (x)) Kn (y) dy


2
2
Z
Z 2
1
1

|f (x y) f (x)| |Kn (y)| dy +


(|f (x y)| + |f (x)|) |Kn (y)| dy
2
2
Z
Z 2
 1
1

<
|Kn (y)| dy + (2M )
dy
2 2
2
4M
 Z 2 

 1
< +
dy = .
2 2 2 0
The above inequality is independent of x, establishing the Theorem.
Corollary 1. Let f (x) satisfy (i) and (ii), and let  > 0. Then there exists a trignometric polynomial
P (x) =

n
X

aj eijx

j=n

(with aj C for each j) such that


sup |P (x) f (x)| .
xR

Proof. Since n (f, x) is a trignometric polynomial, the Corollary follows from Theorem 26 by
taking P (x) = n (f, x) with n sufficiently large.
We now prove Weyls Theorem (Theorem 24). It suffices to show that if R is irrational
and 0 a b 1, then
|{r n : 2{r} [2a, 2b]}|
= b a.
n
n
lim

(13)

Lemma 3. Let R with irrational. Suppose f : R C satisfies (i) and (ii) of the previous
section. Then
Z 2
n
1X
1
lim
f (2r) =
f (x) dx.
n n
2
0
r=1
Proof. Let

1X
1
Gn (f ) =
f (2r)
n r=1
2
44

f (x) dx.
0

We want to show that limn Gn (f ) = 0. First, we consider f (x) = eisx where s is an integer. If
s = 0, then f (x) 1 and
n

1X
1
Gn (f ) = Gn (1) =
1
n r=1
2

dx = 0.
0

Thus, limn Gn (f ) = 0 in the case f (x) = eisx with s = 0. Now, suppose s 6= 0. Then


Z 2
n

1 X
1


2irs
isx
e

e dx
|Gn (f )| =

n
2 0
r=1

n

1 X

2irs
e
=


n r=1


1 2is e2ins 1
=
e
e2is 1
n

2
n|e2is

1|

Thus, in this case, limn Gn (f ) = 0.


P
isx
Now, we consider the case when f (x) = m
(i.e., f (x) is a trignometric polynos=m as e
mial). Then
! !
Z 2 X
n
m
m
1
1X X
2irs
isx
as e

lim Gn (f ) = lim
as e
dx
n
n
n r=1 s=m
2 0
s=m
!
Z 2
n
m
X
1
1 X 2irs
=
as lim
e

eisx dx
n
n
2
0
s=m
r=1
=

m
X
s=m


as lim Gn eisx = 0.
n

Thus, the lemma holds for trignometric polynomials.


We now consider the general case when f (x) satisfies (i) and (ii). Let  > 0. We show that if
n is sufficiently large, then |Gn (f )| < . By the Corollary to Theorem 26, there is a trignometric
polynomial P (x) such that
|f (x) P (x)| <


3

for all real x.

Also, since the lemma has already been established for trignometric polynomials, there is an N

45

such that if n N , then |Gn (P )| < /3. Hence, for all n N ,


|Gn (f )| |Gn (P )| + |Gn (f ) Gn (P )|


Z 2
n


X
 1
1

< +
(f (2r) P (2r))
(f (x) P (x)) dx

3 n r=1
2 0
n


1X
1
+
|f (2r) P (2r)| +
3 n r=1
2
Z 2


1

< + +
dx = ,
3 3 2 0 3

|f (x) P (x)| dx
0

completing the proof.


Proof of Theorem 24. Let  > 0. We will apply Lemma 3 to two functions f+ (x) and f (x)
satisfying (ii). Their precise definitions are not important; a rough graph suffices for the proof. But
since graphs are more difficult to print than precise definitions in TEX, we leave it to the reader to
graph

0
if x [0, 2(a )) [2(b + ), 2)

1
1

x (a )
if x [2(a ), 2a)

f+ (x) = 2

1
if x [2a, 2b)

1
1

x + (b + ) if x [2b, 2(b + ))


2

and

0
if x [0, 2a) [2b, 2)

1
1

x a
if x [2a, 2(a + ))

.
f+ (x) = 2

1
if x [2(a + ), 2(b ))

1 x + 1 b if x [2(b ), 2b)


2

Note that f+ (x) and f (x) are defined for all real numbers x by the above and (ii). Thus,
n
X

f+ (2r) |{r n : 2{r} [2a, 2b]}|

r=1

n
X

f (2r).

r=1

By Lemma 3, there is an N such that if n N , then


n

1X
1
f+ (2r)
n r=1
2

f+ (x) dx +  =

1
(2(b a) + 2) +  = (b a) + 2
2

f (x) dx  =

1
(2(b a) 2)  = (b a) 2.
2

and
n

1X
1
f (2r)
n r=1
2

Z
0

46

Hence, for all n N ,




|{r n : 2{r} [2a, 2b]}|


< 2,

(b

a)


n
from which Theorem 24 follows.
The following are two problems related to this subject. The details of their solutions are omitted
here.
(1) Let d {0, 1, 2, . . . , 9}. What is the proportion of times that 2n begins with the digit d as n
runs through the positive integers? More specifically, compute
|{n x : 2n begins with the digit d}|
.
n
x
lim

(By the way, of the first 1000 values of 2n beginning with n = 0, exactly 301 begin with the digit
1, and log10 2 = 0.3010 . . . .)
(2) (Monthly Problem, 1986) Suppose x R with x > 1. Let an = [xn ] where n is a positive
integer. Let S = 0.a1 a2 a3 . . . . (For example, if x = , then S = 0.393197 . . . since [] = 3,
[ 2 ] = 9, [ 3 ] = 31, and [ 4 ] = 97.) Prove that S is irrational.
Homework:
1. Let k and n be positive integers with n > 2k. Prove that
n1
X
j=0

2k

cos

2j
n

2k
k
22k


=

n.

(Hint: Use the binomial theorem.)


2. Let {1 , 2 , . . . } be a sequence of real numbers. Prove that the sequence is uniformly distributed modulo one if and only if for every a and b with 0 a b 1,
|{r n : {r } (a, b)}|
= b a.
n
n
lim

P
3. Let
aro sum of
j=0 aj be a divergent series with aj > 0 for each j 0. Prove that the Ces`
the series is infinite.
4. Calculate the Ces`aro sum of each of the following series.
(a) 1 1 + 1 1 + 1 1 + .
1 1 1
(b) 1 + + .
2 4 8
5. Prove the following:
47

Theorem 27. Let {k } be a sequence of real numbers satisfying


n

1 X 2imk
e
=0
n n
k=1
lim

for every non-zero integer m. Then {k } is uniformly distributed modulo 1.


(Hint: Look at the proof of Theorem 24 with k replacing k and decide what changes need
to be made. You do not need to rewrite the proofs if you point out clearly where the changes
need to be made and what the changes are.)

6. Prove that the sequence { n}


n=1 is uniformly distributed modulo 1.
7. Prove that the sequence {log n}
n=1 is not uniformly distributed modulo 1.
8. Show that there exists a sequence {aj }
j=0 such that the partial sums sr =
both
Pn
sr
lim inf r=0 =
n
n+1
and
Pn
sr
lim sup r=0 = +.
n+1
n

48

Pr

j=0

aj satisfy

You might also like